df188449ceb744e30fd9c695c1751cc7988bc1b2
[course.git] / latex / problems / Serway_and_Jewett_4 / problem23.22.tex
1 \begin{problem*}{23.22}
2 A rectangular coil with resistance $R$ has $N$ turns, each of length
3 $l$ and width $w$ as shown in Figure P23.22.  The coil moves in a
4 uniform magnetic field \vect{B} with constant velocity $v$.  What are
5 the magnitude and direction of the total magnetic force on the coild
6 as it
7  \Part{a} enters,
8  \Part{b} moves within, and
9  \Part{c} leaves
10  the magnetic field.
11 \end{problem*} % problem 23.22
12
13 \begin{solution}
14 \Part{a}
15 As in Problem 13, $d\Phi_B/dt = wBv$, so the induced current is
16 \begin{align}
17  I = \frac{\varepsilon}{R} = \frac{-d\Phi_B/dt}{R} = \frac{-wvBN}{R} \;,
18 \end{align}
19 where the $-$ sign indicates it is counterclockwise (against the
20 changing flux direction).  The force on the leading wires is
21 \begin{equation}
22  \vect{F} = I\vect{l}\times\vect{B} = -I\cdot Nw\cdot B\ihat
23           = \ans{\frac{-v(wBN)^2}{R}\ihat} \;.
24 \end{equation}
25
26 \Part{b}
27 Once the coil is inside the magnetic field, the flux becomes constant,
28 so there is no induced emf driving a current, and thus \ans{no net
29  force} on the coil.
30
31 \Part{c}
32 The situation here is the inverse of that in \Part{a}, so the induced
33 emf is clockwise, but the current through the portion of loop in the
34 magnetic field is \emph{still up}, so the force is unchanged.
35 \begin{equation}
36  \vect{F} = \ans{\frac{-v(wBN)^2}{R}\ihat}
37 \end{equation}
38 \end{solution}